Dadgarnia

New Member
ارسال ها
1,350
لایک ها
1,127
امتیاز
0
#1
با توجه به اينكه در قسمت جبر ممتاز ماراتني وجود نداره و قسمت جبر هم زياد فعال نيست با اجازه ي جناب math تصميم به ايجاد كردن اين ماراتن گرفتم. قوانين هم ايناست:
١- فقط فارسي بنويسيد.
٢- تا سوال حل نشده اي وجود داره سوال جديدي نذاريد.
٣- هر كس سوالي رو حل كرد بايد سوال بعدي رو خودش بذاره.
٤- سوالات در سطح مرحله دو و بالاتر باشه.
سوال اول رو من ميذارم و اميدوارم اين ماراتن هم رونق بگيره.
تمام توابع
رو بيابيد به طوري كه براي هر
داشته باشيم:
 

darya.f

New Member
ارسال ها
182
لایک ها
114
امتیاز
0
#2
پاسخ : ماراتن جبر (سطح ممتاز)

با توجه به اينكه در قسمت جبر ممتاز ماراتني وجود نداره و قسمت جبر هم زياد فعال نيست با اجازه ي جناب math تصميم به ايجاد كردن اين ماراتن گرفتم. قوانين هم ايناست:
١- فقط فارسي بنويسيد.
٢- تا سوال حل نشده اي وجود داره سوال جديدي نذاريد.
٣- هر كس سوالي رو حل كرد بايد سوال بعدي رو خودش بذاره.
٤- سوالات در سطح مرحله دو و بالاتر باشه.
سوال اول رو من ميذارم و اميدوارم اين ماراتن هم رونق بگيره.
تمام توابع
رو بيابيد به طوري كه براي هر
داشته باشيم:





---- دو نوشته به هم متصل شده است ----

سوال بعد:

F رو از RبهR بىابىد
 

Dadgarnia

New Member
ارسال ها
1,350
لایک ها
1,127
امتیاز
0
#3
پاسخ : ماراتن جبر (سطح ممتاز)




---- دو نوشته به هم متصل شده است ----

سوال بعد:

F رو از RبهR بىابىد
اونجايي كه ميگيد g يك به يكه چجوري اينو بدست آورديد؟ بعدشم اگه g يك به يكه چجوري مي تونه ثابت باشه.
 

darya.f

New Member
ارسال ها
182
لایک ها
114
امتیاز
0
#4
پاسخ : ماراتن جبر (سطح ممتاز)


خب ىنى جواب نداره؟!
اونجايي كه ميگيد g يك به يكه چجوري اينو بدست آورديد؟ بعدشم اگه g يك به يكه چجوري مي تونه ثابت باشه.
 

darya.f

New Member
ارسال ها
182
لایک ها
114
امتیاز
0
#6
پاسخ : ماراتن جبر (سطح ممتاز)

مي دونيم كه
پس اون نتيجه اي كه شما گرفتيد غلطه
اها بله همون اول باىد تو 1-1 ىش اىن شرط رو روش مىذاشتم..درست مىفرماىىد..

---- دو نوشته به هم متصل شده است ----

با توجه به اينكه در قسمت جبر ممتاز ماراتني وجود نداره و قسمت جبر هم زياد فعال نيست با اجازه ي جناب math تصميم به ايجاد كردن اين ماراتن گرفتم. قوانين هم ايناست:
١- فقط فارسي بنويسيد.
٢- تا سوال حل نشده اي وجود داره سوال جديدي نذاريد.
٣- هر كس سوالي رو حل كرد بايد سوال بعدي رو خودش بذاره.
٤- سوالات در سطح مرحله دو و بالاتر باشه.
سوال اول رو من ميذارم و اميدوارم اين ماراتن هم رونق بگيره.
تمام توابع
رو بيابيد به طوري كه براي هر
داشته باشيم:
واسه حالت اول که اشتباه شد:
 

Dadgarnia

New Member
ارسال ها
1,350
لایک ها
1,127
امتیاز
0
#7
پاسخ : ماراتن جبر (سطح ممتاز)

مي دونيم كه
پس اون نتيجه اي كه شما گرفتيد غلطه
اها بله همون اول باىد تو 1-1 ىش اىن شرط رو روش مىذاشتم..درست مىفرماىىد..

---- دو نوشته به هم متصل شده است ----


واسه حالت اول که اشتباه شد:
چجوري بدست آورديد
توي حالت دوم درستش اينه
 

darya.f

New Member
ارسال ها
182
لایک ها
114
امتیاز
0
#9
پاسخ : ماراتن جبر (سطح ممتاز)


تابع رو از RبهR بىابىد
بالا شلوغ شد..اىنجا دوباره گذاشتم..
 

darya.f

New Member
ارسال ها
182
لایک ها
114
امتیاز
0
#10
پاسخ : ماراتن جبر (سطح ممتاز)

راهنماىى:f(0)=c درنظر بگىرىد..و (f(c ، و اىنا رو تشکىل بدىن و c بازى کنىن..نتاىج خوبى بدست مىاد
 

Dadgarnia

New Member
ارسال ها
1,350
لایک ها
1,127
امتیاز
0
#11
پاسخ : ماراتن جبر (سطح ممتاز)


تابع رو از RبهR بىابىد
بالا شلوغ شد..اىنجا دوباره گذاشتم..
فرض کنید رابطه ی بالا
و
باشه. داریم:

اگه
باشه جواب
بدست میاد در غیر این صورت اگر در رابطه ی آخر
را جایگذاری کنیم داریم:





اگر در رابطه ی بالا
را قرار دهیم داریم
و با قرار دادن
در رابطه ی بالا بدست می آید
که در صورت سوال صدق نمی کند پس تنها جواب سوال
است.
سوال بعد:
همه ی چند جمله ای های
را بیابید به طوریکه برای هر سه عدد صحیح
که
،
صحیح باشد.


 
آخرین ویرایش توسط مدیر

math1998

New Member
ارسال ها
336
لایک ها
224
امتیاز
0
#12
پاسخ : ماراتن جبر (سطح ممتاز)

فرض کنید رابطه ی بالا
و
باشه. داریم:

اگه
باشه جواب
بدست میاد در غیر این صورت اگر در رابطه ی آخر
را جایگذاری کنیم داریم:





اگر در رابطه ی بالا
را قرار دهیم داریم
و با قرار دادن
در رابطه ی بالا بدست می آید
که در صورت سوال صدق نمی کند پس تنها جواب سوال
است.
سوال بعد:
همه ی چند جمله ای های
را بیابید به طوریکه برای هر سه عدد صحیح
که
،
صحیح باشد.






اما
بی نهایت مقدار میتونه داشته باشه پس




حالا اگه
رو خیلی بزرگ کنیم داریم :




مقایسه


اخرش رو اشتباه نوشتم ضرایب
منظورم بود.







 
آخرین ویرایش توسط مدیر

AHZolfaghari

Well-Known Member
ارسال ها
935
لایک ها
1,654
امتیاز
93
#13
پاسخ : ماراتن جبر (سطح ممتاز)

سوال بعد رو با اجازتون من میذارم .
n یک عدد طبیعی است . کم ترین مقدار C را بیابید که برای n عدد نامنفی


داشته باشیم :

توجه کنید که i و j متمایزند .​
 

math1998

New Member
ارسال ها
336
لایک ها
224
امتیاز
0
#14
پاسخ : ماراتن جبر (سطح ممتاز)

سوال بعد رو با اجازتون من میذارم .
n یک عدد طبیعی است . کم ترین مقدار C را بیابید که برای n عدد نامنفی


داشته باشیم :

توجه کنید که i و j متمایزند .​
جواب
درسته ؟
 

darya.f

New Member
ارسال ها
182
لایک ها
114
امتیاز
0
#15
پاسخ : ماراتن جبر (سطح ممتاز)

سوال بعد رو با اجازتون من میذارم .
n یک عدد طبیعی است . کم ترین مقدار C را بیابید که برای n عدد نامنفی


داشته باشیم :

توجه کنید که i و j متمایزند .​
ببخشىد جوابش
نىست؟

---- دو نوشته به هم متصل شده است ----

سوال بعد رو با اجازتون من میذارم .
n یک عدد طبیعی است . کم ترین مقدار C را بیابید که برای n عدد نامنفی


داشته باشیم :

توجه کنید که i و j متمایزند .​
ببخشىد جوابش
نىست؟
 

AHZolfaghari

Well-Known Member
ارسال ها
935
لایک ها
1,654
امتیاز
93
#16
پاسخ : ماراتن جبر (سطح ممتاز)

نه متاسفانه هر دو جواب نادرسته [MENTION=17012]darya.f[/MENTION] [MENTION=18610]math1998[/MENTION]
 

math1998

New Member
ارسال ها
336
لایک ها
224
امتیاز
0
#17
پاسخ : ماراتن جبر (سطح ممتاز)

سوال بعد رو با اجازتون من میذارم .
n یک عدد طبیعی است . کم ترین مقدار C را بیابید که برای n عدد نامنفی


داشته باشیم :

توجه کنید که i و j متمایزند .​

این راه حله رو زیاد مطمئن نیستم اما حالا ببین درسته یا نه!!!

نامساوی کاملا همگنه پس میتونیم شرط
حالا فرض کنیم
تا از اعداد ناصفرند پس اگر هیچکدام از
صفر نباشند در عبارت میان خوب پس باید ثابت کنیم


حالا با استفاده از مورهد داریم :


ما میخوایم
مینیمم باشه پس عبارت سمت چپ باید مینیمم باشه

پس
عدد صفر
تا ناصفر مثل
داریم :

حالا دیگه ساده شد با دوتا متغیر راحت داریم که مینیمم
برابر
اما ضرب دو عدد با جمح ثابت وقتی مینیممه که دوتا تا حد امکان به هم نزدیک باشن
یعنی


پس داریم
 

AHZolfaghari

Well-Known Member
ارسال ها
935
لایک ها
1,654
امتیاز
93
#18
پاسخ : ماراتن جبر (سطح ممتاز)

این راه حله رو زیاد مطمئن نیستم اما حالا ببین درسته یا نه!!!نامساوی کاملا همگنه پس میتونیم شرط
حالا فرض کنیم
تا از اعداد ناصفرند پس اگر هیچکدام از
صفر نباشند در عبارت میان خوب پس باید ثابت کنیم
حالا با استفاده از مورهد داریم :​
ما میخوایم
مینیمم باشه پس عبارت سمت چپ باید مینیمم باشه​
پس
عدد صفر
تا ناصفر مثل
داریم :حالا دیگه ساده شد با دوتا متغیر راحت داریم که مینیمم
برابر
اما ضرب دو عدد با جمح ثابت وقتی مینیممه که دوتا تا حد امکان به هم نزدیک باشنیعنی
پس داریم
حقیقتش من نامساوی مورهد رو نمیدونم چیه و استفادش چجوریه و ... کلا نمیدونم !!من اول دو تا 1 دادم و بقیه رو صفر بدست اومده C از 1/8 بزرگ تر مساوی و سعی کردم برای 1/8 اثبات کنم :بقیه اگه بلدن باید چک کنن که درسته یا نه . من شخصا اول که با توجه به همگنی فرض کردم مجموعشون یکه . بعد از میکسینگ استفاده کردم . مثلا x_1 , x_2 رو گرفتم یکیشونو صفر کردم و اون یکی رو برابر جمع دو تاشون گذاشتم . با این کار فرضیات که بهشون دست نمیخوره . اگه بنویسید حکم رو هم بزرگ میکنه . پس همین جوری یکی یکی صفر میکنیم تا حالت دوتایی . حالت دوتایی هم که معلومه و با حسابی هندسی اثبات میشه .

---- دو نوشته به هم متصل شده است ----

سوال بعد :
آیا تابع f از اعداد طبیعی به خودش وجود دارد بطوریکه به ازای هر n طبیعی داشته باشیم

f(f(n)) = n + 1
 

Dadgarnia

New Member
ارسال ها
1,350
لایک ها
1,127
امتیاز
0
#19
پاسخ : ماراتن جبر (سطح ممتاز)

سوال بعد :
آیا تابع f از اعداد طبیعی به خودش وجود دارد بطوریکه به ازای هر n طبیعی داشته باشیم

f(f(n)) = n + 1
اگه از دو طرف رابطه ي بالا f بگيريم بدست مياد:
و اگه در صورت سوال
رو قرار بديم بدست مياد:


حالا با استفاده از دو رابطه ي بدست اومده مي تونيم با استقرا ثابت كنيم
اگه اين رابطه رو توي صورت سوال جاگذاري كنيم داريم
كه تناقضه پس چنين تابعي وجود نداره.
سوال بعد:
فرض کنید
یک چند جمله ای با ضرایب صحیح باشد که برای بعضی از اعداد صحیح نا صفر
است. نشان دهید
برای هر عدد حقیقی ناصفر.
 
آخرین ویرایش توسط مدیر

Dadgarnia

New Member
ارسال ها
1,350
لایک ها
1,127
امتیاز
0
#20
پاسخ : ماراتن جبر (سطح ممتاز)

برای ادامه دادن به ماراتن من سوال بعد رو می ذارم هر کی سوال قبل رو حل کرد جوابش رو همین جا بذاره:
تمام توابع
را بیابید به طوریکه برای هر
داشته باشیم:
 
بالا